Difference between revisions of "2021 AMC 10B Problems/Problem 4"

(Redirected page to 2021 AMC 12B Problems/Problem 2)
(Tag: New redirect)
Line 1: Line 1:
At a math contest, <math>57</math> students are wearing blue shirts, and another <math>75</math> students are wearing yellow shirts. The 132 students are assigned into <math>66</math> pairs. In exactly <math>23</math> of these pairs, both students are wearing blue shirts. In how many pairs are both students wearing yellow shirts?
+
#redirect [[2021 AMC 12B Problems/Problem 2]]
 
 
<math>\textbf{(A)} ~23 \qquad\textbf{(B)} ~32 \qquad\textbf{(C)} ~37 \qquad\textbf{(D)} ~41 \qquad\textbf{(E)} ~64</math>
 

Revision as of 17:49, 11 February 2021